tzyc
Thanks Received: 0
Atticus Finch
Atticus Finch
 
Posts: 323
Joined: May 27th, 2012
 
 
trophy
Most Thankful
 

Q4 - Mayor of Plainsville: In order

by tzyc Mon Mar 04, 2013 9:55 pm

Hello,

I'm a little not sure what "cause" means in this question...
Does it mean ideal, purpose etc instead of the normal usage such as causing some problems...?
so pursue a cause→advance the cause farthest
And citizen's group says
~advance the cause farthest (did not choose to build a new business park)→~pursue a cause (has other interests)
is this correct?

Thank you
User avatar
 
tommywallach
Thanks Received: 468
Atticus Finch
Atticus Finch
 
Posts: 1041
Joined: August 11th, 2009
 
This post thanked 1 time.
 
 

Re: Q4 - Mayor of Plainsville: In order

by tommywallach Wed Mar 06, 2013 9:24 pm

Hey Strawberry,

Did you look it up in the dictionary? That may sound like a facetious question, but I really don't mean it that way. Anytime you see a word used in a strange way (or a word you don't know), make sure to look it up. Words have a tendency to repeat on the LSAT; the vocabulary is relatively small.

The definition of cause that applies here is:

2 a principle, aim, or movement that, because of a deep commitment, one is prepared to defend or advocate: she devoted her life to the cause of deaf people | I'm raising money for a good cause .

I'm sure you've heard a sentence like "It's for a good cause."

So I think your take on the question is mistaken, but that's just because you didn't understand the meaning they were using.

Citizen's Group:

Conclusion: Mayor has interests other than economy in mind
Premise: Business park better for economy than highway

The gap here is that the Mayor must have other interests just because there are better plans that the one he's pursuing.

(A) is the answer because it means the Mayor can't really be pursuing the cause (helping the economy) because he isn't doing the thing that would be best vis-a-vis that cause.

(B) is way too vague. We can't speak about any goal. Besides, we don't know if the business park would require public revenu.es

(C) is totally out of scope. Nowhere does it say that anybody has to be consulted about anything!

(D) talks about the "worth" of a given cause. But we never discuss whether improving the economy of Plainsville is a "worthy" cause.

(E) brings in action from the community, which is irrelevant here.

Hope that helps!

-t
Tommy Wallach
Manhattan LSAT Instructor
twallach@manhattanprep.com
Image
 
sarahv
Thanks Received: 0
Vinny Gambini
Vinny Gambini
 
Posts: 2
Joined: August 14th, 2014
 
 
 

Re: Q4 - Mayor of Plainsville: In order

by sarahv Thu Aug 14, 2014 3:53 pm

How can we assume that the mayor knows that the business park is the best way to improve the economy? Don't we need to assume this to select A?
 
bd8112
Thanks Received: 0
Vinny Gambini
Vinny Gambini
 
Posts: 2
Joined: February 13th, 2014
 
 
 

Re: Q4 - Mayor of Plainsville: In order

by bd8112 Fri Sep 12, 2014 3:41 pm

Sarah's right! (A) is the only plausible answer to this question, so you really couldn't challenge it, but it has a glaring oblique context problem. Just because the Citizens' group states that the new business park would bring in twice the business that the highway would, we absolutely cannot infer that the Mayor believes this to be the case. What's the deal, LSAC? How is this answer justifiable?
 
asafezrati
Thanks Received: 6
Atticus Finch
Atticus Finch
 
Posts: 116
Joined: December 07th, 2014
 
 
 

Re: Q4 - Mayor of Plainsville: In order

by asafezrati Wed Sep 23, 2015 3:13 pm

bd8112 Wrote:Sarah's right! (A) is the only plausible answer to this question, so you really couldn't challenge it, but it has a glaring oblique context problem. Just because the Citizens' group states that the new business park would bring in twice the business that the highway would, we absolutely cannot infer that the Mayor believes this to be the case. What's the deal, LSAC? How is this answer justifiable?

We don't know that the mayor "knows." But this question isn't a sufficient assumption question, it's a principle strengthen question. The other answers are totally wrong, and this one is close to the reasoning in the stimulus.